The Stacks project

9.26 Transcendence

We recall the standard definitions.

Definition 9.26.1. Let $K/k$ be a field extension.

  1. A collection of elements $\{ x_ i\} _{i \in I}$ of $K$ is called algebraically independent over $k$ if the map

    \[ k[X_ i; i\in I] \longrightarrow K \]

    which maps $X_ i$ to $x_ i$ is injective.

  2. The field of fractions of a polynomial ring $k[x_ i; i \in I]$ is denoted $k(x_ i; i\in I)$.

  3. A purely transcendental extension of $k$ is any field extension $K/k$ isomorphic to the field of fractions of a polynomial ring over $k$.

  4. A transcendence basis of $K/k$ is a collection of elements $\{ x_ i\} _{i \in I}$ which are algebraically independent over $k$ and such that the extension $K/k(x_ i; i\in I)$ is algebraic.

Example 9.26.2. The field $\mathbf{Q}(\pi )$ is purely transcendental because $\pi $ isn't the root of a nonzero polynomial with rational coefficients. In particular, $\mathbf{Q}(\pi ) \cong \mathbf{Q}(x)$.

Lemma 9.26.3. Let $E/F$ be a field extension. A transcendence basis of $E$ over $F$ exists. Any two transcendence bases have the same cardinality.

Proof. Let $A$ be an algebraically independent subset of $E$. Let $G$ be a subset of $E$ containing $A$ that generates $E/F$. We claim we can find a transcendence basis $B$ such that $A \subset B \subset G$. To prove this, consider the collection $\mathcal{B}$ of algebraically independent subsets whose members are subsets of $G$ that contain $A$. Define a partial ordering on $\mathcal{B}$ using inclusion. Then $\mathcal{B}$ contains at least one element $A$. The union of the elements of a totally ordered subset $T$ of $\mathcal{B}$ is an algebraically independent subset of $E$ over $F$ since any algebraic dependence relation would have occurred in one of the elements of $T$ (since polynomials only involve finitely many variables). The union also contains $A$ and is contained in $G$. By Zorn's lemma, there is a maximal element $B \in \mathcal{B}$. Now we claim $E$ is algebraic over $F(B)$. This is because if it wasn't then there would be an element $f \in G$ transcendental over $F(B)$ since $F(G) = E$. Then $B \cup \{ f\} $ would be algebraically independent contradicting the maximality of $B$. Thus $B$ is our transcendence basis.

Let $B$ and $B'$ be two transcendence bases. Without loss of generality, we can assume that $|B'| \leq |B|$. Now we divide the proof into two cases: the first case is that $B$ is an infinite set. Then for each $\alpha \in B'$, there is a finite set $B_{\alpha } \subset B$ such that $\alpha $ is algebraic over $F(B_{\alpha })$ since any algebraic dependence relation only uses finitely many indeterminates. Then we define $B^* = \bigcup _{\alpha \in B'} B_{\alpha }$. By construction, $B^* \subset B$, but we claim that in fact the two sets are equal. To see this, suppose that they are not equal, say there is an element $\beta \in B \setminus B^*$. We know $\beta $ is algebraic over $F(B')$ which is algebraic over $F(B^*)$. Therefore $\beta $ is algebraic over $F(B^*)$, a contradiction. So $|B| \leq |\bigcup _{\alpha \in B'} B_{\alpha }|$. Now if $B'$ is finite, then so is $B$ so we can assume $B'$ is infinite; this means

\[ |B| \leq |\bigcup \nolimits _{\alpha \in B'} B_{\alpha }| = |B'| \]

because each $B_\alpha $ is finite and $B'$ is infinite. Therefore in the infinite case, $|B| = |B'|$.

Now we need to look at the case where $B$ is finite. In this case, $B'$ is also finite, so suppose $B = \{ \alpha _1, \ldots , \alpha _ n\} $ and $B' = \{ \beta _1, \ldots , \beta _ m\} $ with $m \leq n$. We perform induction on $m$: if $m = 0$ then $E/F$ is algebraic so $B = \emptyset $ so $n = 0$. If $m > 0$, there is an irreducible polynomial $f \in F[x, y_1, \ldots , y_ n]$ such that $f(\beta _1, \alpha _1, \ldots , \alpha _ n) = 0$ and such that $x$ occurs in $f$. Since $\beta _1$ is not algebraic over $F$, $f$ must involve some $y_ i$ so without loss of generality, assume $f$ uses $y_1$. Let $B^* = \{ \beta _1, \alpha _2, \ldots , \alpha _ n\} $. We claim that $B^*$ is a basis for $E/F$. To prove this claim, we see that we have a tower of algebraic extensions

\[ E/ F(B^*, \alpha _1) / F(B^*) \]

since $\alpha _1$ is algebraic over $F(B^*)$. Now we claim that $B^*$ (counting multiplicity of elements) is algebraically independent over $F$ because if it weren't, then there would be an irreducible $g\in F[x, y_2, \ldots , y_ n]$ such that $g(\beta _1, \alpha _2, \ldots , \alpha _ n) = 0$ which must involve $x$ making $\beta _1$ algebraic over $F(\alpha _2, \ldots , \alpha _ n)$ which would make $\alpha _1$ algebraic over $F(\alpha _2, \ldots , \alpha _ n)$ which is impossible. So this means that $\{ \alpha _2, \ldots , \alpha _ n\} $ and $\{ \beta _2, \ldots , \beta _ m\} $ are bases for $E$ over $F(\beta _1)$ which means by induction, $m = n$. $\square$

Definition 9.26.4. Let $K/k$ be a field extension. The transcendence degree of $K$ over $k$ is the cardinality of a transcendence basis of $K$ over $k$. It is denoted $\text{trdeg}_ k(K)$.

Lemma 9.26.5. Let $L/K/k$ be field extensions. Then

\[ \text{trdeg}_ k(L) = \text{trdeg}_ K(L) + \text{trdeg}_ k(K). \]

Proof. Choose a transcendence basis $A \subset K$ of $K$ over $k$. Choose a transcendence basis $B \subset L$ of $L$ over $K$. Then it is straightforward to see that $A \cup B$ is a transcendence basis of $L$ over $k$. $\square$

Example 9.26.6. Consider the field extension $\mathbf{Q}(e, \pi )$ formed by adjoining the numbers $e$ and $\pi $. This field extension has transcendence degree at least $1$ since both $e$ and $\pi $ are transcendental over the rationals. However, this field extension might have transcendence degree $2$ if $e$ and $\pi $ are algebraically independent. Whether or not this is true is unknown and whence the problem of determining $\text{trdeg}(\mathbf{Q}(e, \pi ))$ is open.

Example 9.26.7. Let $F$ be a field and $E = F(t)$. Then $\{ t\} $ is a transcendence basis since $E = F(t)$. However, $\{ t^2\} $ is also a transcendence basis since $F(t)/F(t^2)$ is algebraic. This illustrates that while we can always decompose an extension $E/F$ into an algebraic extension $E/F'$ and a purely transcendental extension $F'/F$, this decomposition is not unique and depends on choice of transcendence basis.

Example 9.26.8. Let $X$ be a compact Riemann surface. Then the function field $\mathbf{C}(X)$ (see Example 9.3.6) has transcendence degree one over $\mathbf{C}$. In fact, any finitely generated extension of $\mathbf{C}$ of transcendence degree one arises from a Riemann surface. There is even an equivalence of categories between the category of compact Riemann surfaces and (non-constant) holomorphic maps and the opposite of the category of finitely generated extensions of $\mathbf{C}$ of transcendence degree $1$ and morphisms of $\mathbf{C}$-algebras. See [Forster].

There is an algebraic version of the above statement as well. Given an (irreducible) algebraic curve in projective space over an algebraically closed field $k$ (e.g. the complex numbers), one can consider its “field of rational functions”: basically, functions that look like quotients of polynomials, where the denominator does not identically vanish on the curve. There is a similar anti-equivalence of categories (Algebraic Curves, Theorem 53.2.6) between smooth projective curves and non-constant morphisms of curves and finitely generated extensions of $k$ of transcendence degree one. See [H].

Definition 9.26.9. Let $K/k$ be a field extension.

  1. The algebraic closure of $k$ in $K$ is the subfield $k'$ of $K$ consisting of elements of $K$ which are algebraic over $k$.

  2. We say $k$ is algebraically closed in $K$ if every element of $K$ which is algebraic over $k$ is contained in $k$.

Lemma 9.26.10. Let $k'/k$ be a finite extension of fields. Let $k'(x_1, \ldots , x_ r)/k(x_1, \ldots , x_ r)$ be the induced extension of purely transcendental extensions. Then $[k'(x_1, \ldots , x_ r) : k(x_1, \ldots , x_ r)] = [k' : k] < \infty $.

Proof. By multiplicativity of degrees of extensions (Lemma 9.7.7) it suffices to prove this when $k'$ is generated by a single element $\alpha \in k'$ over $k$. Let $f \in k[T]$ be the minimal polynomial of $\alpha $ over $k$. Then $k'(x_1, \ldots , x_ r)$ is generated by $\alpha , x_1, \ldots , x_ r$ over $k$ and hence $k'(x_1, \ldots , x_ r)$ is generated by $\alpha $ over $k(x_1, \ldots , x_ r)$. Thus it suffices to show that $f$ is still irreducible as an element of $k(x_1, \ldots , x_ r)[T]$. We only sketch the proof. It is clear that $f$ is irreducible as an element of $k[x_1, \ldots , x_ r, T]$ for example because $f$ is monic as a polynomial in $T$ and any putative factorization in $k[x_1, \ldots , x_ r, T]$ would lead to a factorization in $k[T]$ by setting $x_ i$ equal to $0$. By Gauss' lemma we conclude. $\square$

Lemma 9.26.11. Let $K/k$ be a finitely generated field extension. The algebraic closure of $k$ in $K$ is finite over $k$.

Proof. Let $x_1, \ldots , x_ r \in K$ be a transcendence basis for $K$ over $k$. Then $n = [K : k(x_1, \ldots , x_ r)] < \infty $. Suppose that $k \subset k' \subset K$ with $k'/k$ finite. In this case $[k'(x_1, \ldots , x_ r) : k(x_1, \ldots , x_ r)] = [k' : k] < \infty $, see Lemma 9.26.10. Hence

\[ [k' : k] = [k'(x_1, \ldots , x_ r) : k(x_1, \ldots , x_ r)] \leq [K : k(x_1, \ldots , x_ r)] = n. \]

In other words, the degrees of finite subextensions are bounded and the lemma follows. $\square$


Comments (9)

Comment #2784 by Reimundo Heluani on

There's a small typo in the proof of 9.26.3 E(G)=F should read F(G)=E (second to last line in first paragraph of the proof).

Comment #4974 by Konstantin on

In the 3rd paragraph of the proof of Lemma 030F one should assume that {\alpha_1,...,\alpha_k} is a minimal subset of B such that \beta_1 is algebraic over F(\alpha_1,...,\alpha_k), in order to make sure that \alpha_1 is algebraic over F(\beta_1,\alpha_2,...,\alpha_n). Otherwise you could be unlucky when \beta_1 happens to be equal to \alpha_2.

Comment #5031 by YW on

In the last paragraph of Lemma 030F, it seems the tower is E/F(B∗)/F(B∗,α1).

Comment #5222 by on

@#4974: No, I think the argument is correct as it stands. Namely, the unlucky scenario you suggest we would have which can't happen as occurs in .

@#5031: No, when we say we have a tower of fields then this means contains and contains . This would not be the case with your suggestion.

Comment #5349 by on

In the fourth paragraph of the proof of Lemma 030F you should say that B * is algebraically independent over F, not on E.

Comment #6293 by Meng-Gen Tsai on

Typo: 'wold' should be 'would' (in the first paragraph of the proof of Lemma 030F.)


Post a comment

Your email address will not be published. Required fields are marked.

In your comment you can use Markdown and LaTeX style mathematics (enclose it like $\pi$). A preview option is available if you wish to see how it works out (just click on the eye in the toolbar).

Unfortunately JavaScript is disabled in your browser, so the comment preview function will not work.

All contributions are licensed under the GNU Free Documentation License.




In order to prevent bots from posting comments, we would like you to prove that you are human. You can do this by filling in the name of the current tag in the following input field. As a reminder, this is tag 030D. Beware of the difference between the letter 'O' and the digit '0'.